1 1 1 1 1 1dl.keywin.org/f/6/f6aa09cee96d90f76619d51be7e69eba.pdf · 2012-04-16 · get prepped!...

30
Get Prepped! PrepTest 27 1 PrepTest 27 (280-287) 1 1 1 1 1 1 Analysis of the Questions: 1. Powell- This is a point at issue question. The first speaker makes a conclusion, the second speaker disagrees. What are they disagreeing about? Do not confuse the first speakers conclusion with either a fact or a premise. Powells conclusion is that private companies are more efficient. (A) This answer is incorrect because Freeman did not dispute the figure of 60%. (B) * This answer is correct; it captures both sides of the dispute. Powell concludes that they are more efficient. Freeman argues that they spend less because they avoid serving high-cost customers. (C) This answer is incorrect because it captures only part of the dispute (Freemans argument). (D) This answer is obviously incorrect; it contains new information that neither side mentioned. Remember to stay within the passage. (E) This answer is incorrect because it captures only one side of the dispute (Freemans argument). 2. Although 90 percent- This is a flawed reasoning question. These are similar to, but not the same as questions that ask you to weaken the argument. You must identify the authors mistake. There are four common types of flaws, which you are probably familiar with by now, if not review them in the logical reasoning lesson. It helps to paraphrase the passage into an argument with premises and a conclusion. Once in the form of an argument, the mistake from the passage becomes more obvious. You must identify a key piece of information, which the author failed to establish in support of his or her argument. Try and paraphrase the passage into a logical argument such as the following: Only 20 percent of all persons are able to understand a news story about DNA. Therefore, 80 percent of all persons cannot make well-informed medical choices or good health-care policy decisions. The author skipped an important premise. He or she did not link the ability to understand a news story about DNA and the ability to make well-informed medical choices. He is comparing apples to oranges. The correct answer is the one that supplies the missing link between an understanding of DNA and the ability to make good medical and policy decisions. (A) This answer is incorrect; it is not a piece of evidence that the author failed to demonstrate. This answer may be valid conclusion based on the passage, but it certainly is not an overlooked premise. (B) This answer is incorrect; it does not address the information that the author overlooked. This is an example where the test writers have worded an answer choice similarly to the passage in an attempt to make it look appealing. As usual, this sort of answer is not correct. It is a faulty conclusion made to look like the right answer. (C) This answer is incorrect; it does not provide the missing premise to link an understanding of DNA and the ability to make medical choices. (D) * This answer is correct because it supplies the missing premise, or logical link, that the author had overlooked. If the author had shown this connection, then his or her conclusion would have been valid. (E) This answer is obviously incorrect; it is another example of an incorrect answer that the test writers tried to make appealing. 3. In Yusakawas- This is a make a conclusion question; they are very common. You must make a conclusion based on a given set of facts or conditions. Here, unlike an identify a conclusion type question, the answer is not stated in the passage. When answering these questions, there are several things to keep in mind. Limit your conclusion to the given facts and avoid making assumptions. Do not make a conclusion that is logically possible, but not directly supported by the facts from the passage. Do not select an answer that simply restates a fact or condition (remember, the conclusion is not in the passage). When (as sometimes happens) you find two feasible answer choices, select the one that is narrowest in scope. Note, this is a two-question passage. On the test, you may want to read the passage a second time to answer the second question. Identify the inference supported by the facts from the passage. (A) This answer is incorrect; the passage did not support this inference. The facts of the passage suggested the opposite that there is a relation between size and age. (B) This answer is incorrect; the passage provides no support for this inference. The month-long study did not compare the survival rate of large and small birds of the same age. (C) This is an incorrect answer; the passage tended to refute this inference by

Upload: others

Post on 04-Apr-2020

0 views

Category:

Documents


0 download

TRANSCRIPT

Page 1: 1 1 1 1 1 1dl.keywin.org/f/6/f6aa09cee96d90f76619d51be7e69eba.pdf · 2012-04-16 · Get Prepped! PrepTest 27 3 revenue is bankrupting the most popular magazines. But readership is

Get Prepped! PrepTest 27

1

PrepTest 27 (280-287)

1 1 1 1 1 1 Analysis of the Questions: 1. Powell- This is a point at issue question. The first speaker makes a conclusion, the second speaker disagrees. What are they disagreeing about? Do not confuse the first speaker�s conclusion with either a fact or a premise. Powell�s conclusion is that private companies are more efficient. (A) This answer is incorrect because Freeman did not dispute the figure of 60%. (B) * This answer is correct; it captures both sides of the dispute. Powell concludes that they are more efficient. Freeman argues that they spend less because they avoid serving high-cost customers. (C) This answer is incorrect because it captures only part of the dispute (Freeman�s argument). (D) This answer is obviously incorrect; it contains new information that neither side mentioned. Remember to stay within the passage. (E) This answer is incorrect because it captures only one side of the dispute (Freeman�s argument). 2. Although 90 percent- This is a flawed reasoning question. These are similar to, but not the same as questions that ask you to weaken the argument. You must identify the author�s mistake. There are four common types of flaws, which you are probably familiar with by now, if not review them in the logical reasoning lesson. It helps to paraphrase the passage into an argument with premises and a conclusion. Once in the form of an argument, the mistake from the passage becomes more obvious. You must identify a key piece of information, which the author failed to establish in support of his or her argument. Try and paraphrase the passage into a logical argument such as the following: Only 20 percent of all persons are able to understand a news story about DNA. Therefore, 80 percent of all persons cannot make well-informed medical choices or good health-care policy decisions. The author skipped an important premise. He or she did not link the ability to understand a news story about DNA and the ability to make well-informed medical choices. He is comparing apples to oranges. The correct answer is the one that supplies the missing link between an understanding of DNA and the ability to make good medical and policy decisions. (A) This answer is incorrect; it is not a piece of evidence that the author failed to demonstrate. This answer may be valid conclusion based on the passage, but it certainly is not an overlooked premise. (B) This answer is incorrect; it does not address the information that the author overlooked. This is an example where the test writers have worded an answer choice similarly to the passage in an attempt to make it look appealing. As usual, this sort of answer is not correct. It is a faulty conclusion made to look like the right answer. (C) This answer is incorrect; it does not provide the missing premise to link an understanding of DNA and the ability to make medical choices. (D) * This answer is correct because it supplies the missing premise, or logical link, that the author had overlooked. If the author had shown this connection, then his or her conclusion would have been valid. (E) This answer is obviously incorrect; it is another example of an incorrect answer that the test writers tried to make appealing. 3. In Yusakawa�s- This is a make a conclusion question; they are very common. You must make a conclusion based on a given set of facts or conditions. Here, unlike an identify a conclusion type question, the answer is not stated in the passage. When answering these questions, there are several things to keep in mind. Limit your conclusion to the given facts and avoid making assumptions. Do not make a conclusion that is logically possible, but not directly supported by the facts from the passage. Do not select an answer that simply restates a fact or condition (remember, the conclusion is not in the passage). When (as sometimes happens) you find two feasible answer choices, select the one that is narrowest in scope. Note, this is a two-question passage. On the test, you may want to read the passage a second time to answer the second question. Identify the inference supported by the facts from the passage. (A) This answer is incorrect; the passage did not support this inference. The facts of the passage suggested the opposite� that there is a relation between size and age. (B) This answer is incorrect; the passage provides no support for this inference. The month-long study did not compare the survival rate of large and small birds of the same age. (C) This is an incorrect answer; the passage tended to refute this inference by

Page 2: 1 1 1 1 1 1dl.keywin.org/f/6/f6aa09cee96d90f76619d51be7e69eba.pdf · 2012-04-16 · Get Prepped! PrepTest 27 3 revenue is bankrupting the most popular magazines. But readership is

Get Prepped! PrepTest 27

2

suggesting that a difference in age, even among the same-sized birds, probably indicates a difference in chances of survival. (D) * This is the correct answer. The passage suggested that age, not size, was the determinative factor in chances of survival. However, the correlation between size and survival rate may actually have stemmed from the correlation between size and age. (E) This answer is incorrect; the passage provided no support for an inference that sample size affected the validity of the results. The passage would not support this conclusion, it contradicts it. 4. In Yasukawa�s- This is a strengthening question. The LSAT tests the ability to strengthen an argument because it is an important skill to law students and lawyers. Your ability to handle these questions efficiently is central to doing well on the test; so practice, practice, and practice them some more. Ignore issues and ideas that are irrelevant to strengthening the argument. To answer these questions correctly, you must understand the author�s argument and its conclusion. Be able to paraphrase the argument and then marshal evidence for the author�s position. After a bit of practice, finding the key piece of evidence that strengthens the argument will become second nature. In this instance, the correct answer weakens the critic�s argument. (A) * This is the correct answer; it undermines the critics position. The critic�s argument depended on the assumption that the difference in size of the birds was attributable to a difference in age. This new fact demonstrates that the difference in size was attributable to a difference in species. So, it undermines the assumption upon which the critic�s argument depended. (B) This answer is incorrect; where Yasukawa conducted the study is irrelevant to the criticism of the study. (C) This answer is incorrect; the survival chances of other birds are irrelevant to criticism of the study. (D) This answer is incorrect; it fails to address the argument that age, not size, is the determinative factor. Fights are irrelevant. (E) This is an incorrect answer; it fails to undermine the argument that age, not size, is the determinant factor for chances of survival. 5. During the 1980's- This is a strengthening question. Let�s paraphrase the argument: Japanese collectors have acquired many nineteenth-century Impressionist paintings; therefore, they specifically prefer certain aesthetic qualities found in nineteenth-century Impressionist paintings. (A) This is an incorrect answer; it introduces totally irrelevant information. It fails to address the reason why Japanese collectors acquired nineteenth-century Impressionist paintings. (B) This answer is incorrect; it tends to weaken, rather than strengthen, the author�s argument. An expanding economy is an alternative explanation for the Japanese collectors� activity. Economic growth may have led to an increase in art collection overall, and to the collection of art as an investment. (C) * This answer is correct; it strengthens the author�s argument. Japanese collectors prefer certain qualities found in Japanese prints. The Impressionist paintings have the same qualities. Therefore Japanese collectors prefer the Impressionist paintings. (D) This is an incorrect answer; it introduces irrelevant information about the 1960's and 1970's. It fails to address the reason why Japanese collectors acquired so many nineteenth-century Impressionist paintings, during the 1980's. (E) So what? This does not address why Japanese collectors were interested in nineteenth-century Impressionist paintings from Europe. 6. Frankie- This is an argument structure question. Focus on the �big picture� and not the details. Anna�s argument proceeds how? Pay attention to the structure of Anna�s response to Frankie, and her methods of reasoning. (A) This answer is not correct. Anna argued that it would not benefit the gatherers. (B) This is an incorrect answer. While Anna did reject Frankie�s proposal, she did not proceed by reinterpreting his evidence. She used her own evidence. (C) * This answer is correct; it accurately states how Anna organized and presented her argument. Anna showed what would happen if Frankie�s proposal were adopted by comparing the cloudberry gatherers� situation to that of cacao gatherers. (D) This answer is incorrect for two reasons. First, Anna did not propose a general theory. Second, she used a comparison. Anna addressed Frankie�s proposal, not a specific market situation. (E) This answer is off the mark; Anna did not contend that cloudberries were used for similar purposes as cacao. 7. Because of- This is a resolve the discrepancy question. Find the missing piece of information that the author forgot to include to explain the discrepancy; play the mediator. The paradox is that declining ad

Page 3: 1 1 1 1 1 1dl.keywin.org/f/6/f6aa09cee96d90f76619d51be7e69eba.pdf · 2012-04-16 · Get Prepped! PrepTest 27 3 revenue is bankrupting the most popular magazines. But readership is

Get Prepped! PrepTest 27

3

revenue is bankrupting the most popular magazines. But readership is up, and there are more successful magazines now then is normally the case. (A) This answer is incorrect; it explains why advertising revenues have decreased, but it does not explain why there are now more financially successful magazines. (B) This is an incorrect answer; the size of an audience for a television show is irrelevant how certain magazines are doing compared to others. (C) * This answer is correct; it offers a solution to the apparent discrepancy. The most widely read magazines were vulnerable to a decrease in advertising revenue, the other magazines were not. Thus, the decrease in advertising revenue has affected only the most widely read magazines. (D) This is an incorrect answer; it does not explain the apparent discrepancy. If this were true, then all magazines would be on the brink of bankruptcy. The decline in advertising would threaten the most widely read magazines, and the decline in subscriptions would threaten the others. (E) This answer is incorrect. The number of general interest magazines is irrelevant to solving the apparent discrepancy. 8. The gray squirrel- This is a weakening question. The officials concluded that the poison traps would not harm the owls. They based their argument on the fact that the traps are accessible only to gray squirrels and other rodents. (A) This is an incorrect answer; it would strengthen the officials� argument. It fails to show any connection between a poison that kills squirrels, and harming the owl population. (B) * This answer is correct. It calls into question the officials� argument by showing that there is a connection between the proposed poison and harming the owl population. The poison kills rodents, and the owl population depends on rodents for food; therefore, the poison reduces the owl�s food supply. (C) So what? This fails to address the officials� argument. (D) This answer is incorrect; it fails to address the officials� argument about poison. (E) This answer is incorrect; it fails to show that there is a link between the poisoning of the rodents and harm to the owl population. Choice (B) is a better answer, it established a link between poisoning the rodents and the owl population; it showed that owls eat the rodents. 9. Sales manager- This is a make a conclusion question. The manager says that they sold more meals. Accountant says same-store sales dropped. We can conclude that the extra meals were served at new restaurants. (A) * This answer is correct; it must be true. Overall sales are up, but sales are down at every store that was open more than a year. If there were no new stores, then the overall sales would have declined. Therefore, there must be at least one new store to account for the overall increase in sales. (B) This answer is obviously incorrect. Desirability, not competitiveness, was the issue. (C) This answer is incorrect. Desirability, not quality, is the issue. (D) The focus is on total number sold and desirability. We cannot conclude anything about prices. (E) This answer is incorrect. It is impossible to conclude any thing about overall market share without knowing the size of the overall market. The passage stated nothing about the overall market. 10. Sales manager: This is a weakening question. It is the second of two on the same passage, and (true to form) it addresses only the second speaker�s part. To weaken the accountant�s argument, identify a reason for a decline in sales other than decreased desirability. (A) This answer is incorrect; it tends bolster the accountant�s� argument. A change in menu would explain why the meals are less desirable than they were the year before. (B) This answer is incorrect; it does not weaken the argument. A decrease in the desirability of meals is consistent with an overall downward trend in sales. (C) This answer is incorrect; it fails to address the accountant�s argument. It addresses only those stores where sales have increased. (D) This is an incorrect answer; it does not directly address the reason for a decrease in sales at stores open more than one year. (E) * This answer is correct; if true, it would weaken the accountant�s argument. It provides an alternative explanation for why sales have decreased in restaurants open more than a year. The reason for the decrease is geographic and economic and unrelated to the desirability of the meals. 11. A local chemical- This is a parallel reasoning question. These questions are tricky and time consuming. See the explanation to question 26 for a more complete description of how you should approach these questions. Follow these steps: First, place the argument from the passage into a simple form, where its

Page 4: 1 1 1 1 1 1dl.keywin.org/f/6/f6aa09cee96d90f76619d51be7e69eba.pdf · 2012-04-16 · Get Prepped! PrepTest 27 3 revenue is bankrupting the most popular magazines. But readership is

Get Prepped! PrepTest 27

4

structure is more apparent. Second, identify the flaw in the reasoning from the passage. Third, find a similar flaw in reasoning among the answer choices. According to the author, because the chemical plant could have caused sterility in otters, the sterility in the nearby otters must have been caused by the chemical plant. Maybe the ozone hole caused the sterility. The author failed to consider whether there was an alternative explanation for the sterility in the nearby otters. As a criminal defense attorney would say, �Just because my client had the motive, ability, and opportunity to commit the crime, doesn�t mean that my client did commit the crime.� (A) This is an incorrect answer; it exhibits a flaw in reasoning, but is different than the passage. This argument concludes that, because something is possible, it must happen more frequently. The passage concluded that, because it could have caused, it must have caused. (B) * This is the correct answer; it exhibits the same flaw as the passage. According to this argument; because a diet low in calcium could have caused a drop in egg production, the drop in egg production at a local farm must have been caused by a diet low in calcium. Maybe foxes ate the eggs. (C) Undernourished does not equal infection. Not undernourished equals not diseased. The flaw in reasoning is only somewhat similar to the reasoning of the passage. (D) Apes have two defining features. This fossil has one of those, thus, it is an ape. This is an incorrect answer; it contains a flaw in reasoning that is only somewhat similar to the passage. (E) This answer is clearly incorrect. Its� reasoning is not at all similar to the passage. Track equals bear. Since no bear, track is fake. 12. Clothes made from- This is a make a conclusion question. (A) This is an incorrect answer; the passage does not support this conclusion. The passage cited the affect that high temperatures have on natural fibers, but it stated nothing about low temperatures. (B) This is an incorrect answer; the passage stated that all fibers, when washed at high temperatures, return to their original state, whether straight or curly. The fibers in blended clothes are not immune from the effect of high temperatures. (C) This answer is incorrect. Based on the passage, the only thing we know is that clothes made from natural fibers shrink more easily than other clothes. (D) This answer is incorrect; the passage provided no evidence about whether natural fibers, once they have been curled up, can be made straight again. (E) * This answer is correct; it is a conclusion directly supported by the passage. The reason that natural fibers shrink is because they are curly in their original state. Artificial fibers do not shrink. Therefore, artificial fibers must be straight, not curly, in their original state. 13. Problems caused- This is an assumption question. The correct answer will not be explicitly stated within the passage; you must read between the lines. The passage will contain an explicit conclusion. From the conclusion, work backwards to reveal the hidden assumption, the missing premise, or implicit factual basis. The answer is a logical antecedent, or a necessary pre-condition of the author�s conclusion. In other words, the answer is the unidentified road that the author had to take to reach a particular conclusion. Your job is to identify the road. Be leery of an answer choice that merely restates a fact from the passage or an answer choice that introduces a topic not found in the passage. (A) This is an incorrect answer; it introduces a topic not found in the passage. The author based his or her conclusion on the premise that economic output and technical advances are linked to solving waste disposal problems. The author did not mention a system of fines or law enforcement. (B) Tricky, but no. This answer is incorrect; it is unrelated to the author�s argument. The amount of air and water pollutants released by foreign countries outside Country X has no effect on the problems caused by landfills leaching inside Country X. (C) * This is the correct answer; it is an assumption upon which the author�s argument depends. When you work backwards from the stated conclusion to this assumption, it becomes clear that this is correct. The author concluded that Country X�s waste problems should start to diminish in the next few years. For that to happen, the per capita income must exceed $5,000 in the next few years. For the per capita income to exceed $5,000 in the next few years, there must be an increase in industrial development. (D) This answer is incorrect; Country X has already begun the process of industrialization. (E) This answer is incorrect; it is irrelevant to the author�s argument. 14. Critic- This is an argument structure question. Why does the critic discuss the therapeutic success? (A) This is an incorrect answer; the author did not use the statement (re: relative therapeutic success of

Page 5: 1 1 1 1 1 1dl.keywin.org/f/6/f6aa09cee96d90f76619d51be7e69eba.pdf · 2012-04-16 · Get Prepped! PrepTest 27 3 revenue is bankrupting the most popular magazines. But readership is

Get Prepped! PrepTest 27

5

popular theories) to disprove the evidence against popular theories. (B) * This is the correct answer; the author proposes a new standard, success at solving problems, by which to judge a theory. (C) This contradicts the facts; the theories are usually better in solving the problems. They are not usually scientifically better. (D) This answer is incorrect; it shows what the author considered to be the most important aspect of popular theories. (E) This is incorrect. The critic argues that although theories have no explanatory powers, they do work, solving real problems. 15. Tony- This is an A responds to B question. Tony says that the number of violent acts by anarchists are very few compared to other ideologies. Thus, associating anarchy with violence is inaccurate. How does Keisha�s argument play-off of Tony�s? She points out that, as a percentage, there may be more violent anarchists (say 1 in 100) then violent monarchists (say 1 in 1,000,0000). (A) * This answer is correct; Keisha cast doubt on Tony�s conclusion by showing that a comparison of absolute numbers is valid only between populations of roughly the same size. (B) This answer is incorrect; Keisha introduced no evidence. She merely criticized the validity of Tony�s conclusion. (C) This answer is incorrect; Keisha questioned the argument�s conclusion, not its premises. (D) This answer is incorrect; Keisha did not introduce evidence to show that the groups have no significant qualities in common. (E) This answer is incorrect; Keisha did not question the method that Tony used to classify persons as either an anarchist or not an anarchist. 16. Recent research- This is an assumption question. The facts are: change is sudden, not gradual. Classic theory is that it is gradual. Classical theory must be discarded. Therefore, the theory in general must be discarded. The assumption? The classical theory is the same as the general theory. (A) This answer is incorrect; it merely restates a fact from the passage. (B) This answer is incorrect; this restates a fact. (C) Information about cultures does nothing to justify the author�s conclusion. (D) * This answer is correct; when added to the argument it justifies the author�s conclusion. If most of a theory is incorrect, then the entire theory should be discarded. The classical account is most of the theory of sound change. The classical account is incorrect. Therefore, the entire theory of sound change should be discarded. (E) This answer is incorrect. The author based his or her argument on recent research, not historical records. 17. The stable functioning- This is an argument structure question. What role does this element play? Support? Contradiction? Conclusion? Find the element in the passage. Now identify the other elements. The first sentence is the conclusion. The rest of the passage holds the facts the support this conclusion. (A) This is an incorrect answer; although it appears at the end of the passage, it is not the conclusion. The conclusion appears in the first sentence. (B) * This is a correct answer; it accurately describes the role of the statement in the argument. The conclusion of the passage appeared first, and everything that appeared afterwards supported the conclusion. (C) This answer is incorrect; the statement supported the passage�s conclusion. (D) This answer is incorrect; the statement is a premise, not a consequence, of the argument. (E) This answer is incorrect; the statement is a premise, not an illustration/example. 18. Astronauts who- This is another strengthening question, we have seen a large number of these in this section. The facts are: astronauts see motion and don�t feel motion, they get sick. Therefore, conflicting information causes sickness. (A) * Maybe. (B) This answer is incorrect; it neither supports nor weakens the hypothesis. (C) This answer is incorrect; in this example, there is no conflicting information received by the brain. They see and feel the motion. (D) Maybe. (E) This answer is incorrect; it neither strengthens nor weakens the hypothesis. (A) * This would strengthen the hypothesis by providing an additional real-world example of the theory�s application. Like the astronauts�, the brains of passengers in windowless cabins receive conflicting information because they feel, but don�t see, the motion. (D) This answer is incorrect; if true, it would tend to weaken the hypothesis from the passage. According to the hypothesis, people in aisle seats should be more likely to get motion sickness because they can feel the motion, but can�t see it. 19. Pollen and other- This is a make a conclusion question. Gather the facts and take them one step

Page 6: 1 1 1 1 1 1dl.keywin.org/f/6/f6aa09cee96d90f76619d51be7e69eba.pdf · 2012-04-16 · Get Prepped! PrepTest 27 3 revenue is bankrupting the most popular magazines. But readership is

Get Prepped! PrepTest 27

6

further. (A) This answer is incorrect; it is not a conclusion based on the passage. Although histamines do not produce cold symptoms, the passage left open the possibility (silly as it may be) that a cold itself may be caused by pollen and other allergens. (B) This answer is incorrect; it lacks factual foundation. (C) Maybe. (D) This answer is obviously incorrect; it is a mixed-up reading of the facts. (E) * Maybe. (C) Antihistamines block histamine, which causes congestion. But maybe something else causes cold congestion. This seems possible. (E) * This is a logical conclusion based on the passage. Because histamines do not cause cold symptoms, any beneficial effect of antihistamines on colds is unrelated to blocking the action of histamines. So this is a narrower conclusion than (C), which may or may not be true. 20. A poem is- This is a make conclusion question. The facts are long and detailed. The basic facts are: (1) Poem is any work that uses some musical characteristics. (2) A novel does not usually use musical characteristics. (A) This is an obviously incorrect answer; a painting, a sculpture, and a play are works of art. (B) This is an incorrect answer; there are four named musical characteristics; rhyme is only a single possibility for what makes a work of art a poem; it is not determinative. (C) * This is a correct answer; it is a logical, wholly supported, inevitable conclusion of the passage. Any work of art that exploits some of the musical characteristics of language is a poem. If a novel (which is not a Limerick) exploits some of the musical characteristics of language, then it is a poem. (D) This answer is incorrect; a Limerick can�t be poetry, since it is, by definition, not allowed to be art. (E) �Musical characteristics of sound� is a new concept. 21. In order to- This is an identify an assumption question. (A) This answer is incorrect; who cares if it is �recent?� It is not an assumption on which the manufacturers� argument depends. (B) * This answer is correct; the manufacturers� argument is premised on telecommunications being widely available. For the telecommunications equipment to be widely available it must not be available solely to top government officials. (C) This is an incorrect answer; if true, it tends to strengthen the argument; however, it is not an assumption on which the manufacturers� argument depends. (D) This is an incorrect answer; the manufacturers� have to assume only that the equipment would be effective, not the most effective. The red-flag word ruined this choice. (E) This answer is incorrect. It is not an assumption on which the manufacturers� argument depends, it�s a new issue. 22. Some people- This is a make conclusion question. Take the facts and extend them to make a conclusion. Remember the baking analogy? Take the ingredients, mix them, and then bake them to make a cake, not just a bowl of mixed ingredients. Here are the ingredients for this passage: some people believe this thing, all bureaucracies are hierarchies, this department is a bureaucracy, this department operated differently than most bureaucracies. What can we conclude? This department is hierarchical, but different. (A) All we know is that it operates differently than a bureaucracy. This doesn�t mean it operates like a non-bureaucracy. (B) This answer is incorrect; all bureaucracies are hierarchical organizations; but all hierarchical organizations are not bureaucracies. This one mixed-up necessary and sufficient conditions. (C) This answer is incorrect; �nothing� is an overly broad conclusion. The facts did not discuss how hierarchical organizations operate, so we can�t make this conclusion. (D) * This answer is correct; it is a logical conclusion based on the passage. The public works department (DPW) is bureaucratically controlled; therefore, it is hierarchical. The DPW operates in a manner quite differently than other organizations. Therefore, not all bureaucratic (thus, hierarchical) organizations operate in the same way. (E) The facts concede that bureaucracies often operate one way. The DPW does not. 23. Research indicates- This is a flaw in the reasoning question. Do you want to bet it is one of the common flaws? The facts are: 90% of insomniacs drink lots of coffee. Tom drinks lots of coffee. Therefore, Tom is an insomniac. The flaw is obvious. There is a cause and effect mix-up. Simply drinking lots of coffee doesn�t mean you are an insomniac. (A) Although initially attractive, this answer is incorrect. It misstates the passage. The passage stated that 10% of insomniacs do not consume large amounts of coffee. (B) This answer is incorrect; this is not a flaw in the reasoning. (C) * This answer is correct; the passage ignores the fact that insomniacs are probably a minuscule subset of the population of persons who

Page 7: 1 1 1 1 1 1dl.keywin.org/f/6/f6aa09cee96d90f76619d51be7e69eba.pdf · 2012-04-16 · Get Prepped! PrepTest 27 3 revenue is bankrupting the most popular magazines. But readership is

Get Prepped! PrepTest 27

7

consume large amounts of coffee. Most insomniacs are persons who consume large amounts of coffee, but very few persons who consume large amounts of coffee are insomniacs. See, cause and effect. (D) This answer is incorrect. To do this is not a flaw. (E) This is an incorrect answer. The passage said, �quite likely� not �always.� 24. Folklorist- This is a strengthening question. The wording is unusual, but if you think about it, you realize that you are supposed to make the folklorist�s argument stronger. Let�s edit the passage and insert the correct choice. Facts: speech improves memory. With writing everyone gets confused. (Insert new fact: Economy of expression is best thing.) In oral, useless stuff is not discussed. Therefore, oral is better. (A) This answer is incorrect; it misstates the passage. According to the passage, exclusive dependence on speech improves the memory, not accuracy in communications. (B) This answer is incorrect; it does not strengthen the folklorist�s argument. This could be a recommendation of his, but we don�t want a recommendation. (C) This answer is incorrect. Because it does not distinguish between oral and written tradition, it does not strengthen the author�s argument. (D) This answer is correct. The passage concluded that oral traditions are preferable to written ones. Oral traditions do lead to an economy of expression, while written ones lead to verbosity. Therefore, economy of expression is preferable to verbosity. (E) Close, but this answer is incorrect; it does not strengthen the folklorist�s argument. 25. When interviewing- This is a flaw in the reasoning question. It is actually more like a weakening question, since you are asked to identify the new facts that would weaken the conclusion that the managers should not ask about hobbies. (A) This would be a very good reason to ask about hobbies. (B) * This doesn�t seem to be a reason against asking about hobbies. On second thought, it is a good reason why they should not ask about hobbies, and we want the answer that does not weaken the conclusion. (C), (D), & (E) are all good reasons to ask about hobbies, so these would make the conclusion flawed. 26. Researcher- This is a parallel reasoning question. One of the ways by which the LSAT tests your ability to evaluate arguments and methods of reasoning is to ask that you mirror a set of conditions. It is important to practice them because they are difficult and unfamiliar. These questions can be time-consuming traps for the uninitiated test taker. However, once you have practiced, you will be able to handle them, or know how to avoid them. First, eliminate answer choices. Note that all the answer choices except (E) deal with sickness and curing sickness. (E) is about the causes of a sickness. This is different enough from the other four to make us disfavor it. Although there were none here, beware of answer choices that deal with the same subject matter, or incorporate the same wording as the passage; generally, they are red herrings. Second, we try to grasp the argument. It is often easiest to identify patterns of reasoning when the arguments are in diagram form. So, we use words or symbols to make a diagram form. You are only concerned with the blueprint, or design, of the arguments. The right answer is not necessarily identical to the passage, but it is the most similar in its reasoning pattern. In this particular problem the argument doesn�t lend itself to a symbol diagram, so paraphrase it. Most dreams don�t resemble future events. So it is silly to think one has ESP just because a few dreams did resemble future events. (A) Close. But the second sentence of this argument does not closely match the first sentence of the passage. (B) No. This one says that surgery works for some, but not others. (C) This isn�t even close. It mentions �other factors.� (D) * This answer is correct; its pattern of reasoning is similar to the passage. While it is not identical to the passage, it more closely fits the pattern of reasoning than the other answer choices. Most aspirin takers don�t die. So it is silly to think aspirin kills people just because some aspirin takers do die early. (E) This one has a shift, from �children� to �people.� Two things were discussed, but we only want to see one.

Page 8: 1 1 1 1 1 1dl.keywin.org/f/6/f6aa09cee96d90f76619d51be7e69eba.pdf · 2012-04-16 · Get Prepped! PrepTest 27 3 revenue is bankrupting the most popular magazines. But readership is

Get Prepped! PrepTest 27

8

(288-291)

2 2 2 2 2 2 Question Set Difficulty:

Sets one and four were the easiest sets in this section. Set three is quite difficult, since not much information is given to aid in understanding the interrelationships. As such, one is often pushed toward working backward using the answer choices. Set two is somewhat challenging, since you are tracking multiple things. Overview of puzzle set #1- Building Inspectors

Over a seven-day period, seven building inspectors (F/G/H/J/K/L/M) shall each inspect the building one time. As they cannot view the site together, a simple line diagram is all that we need. (Figure 1)

Condition 1: F = 3 or 5 Condition 2: L ≠ 4 nor 6 Condition 3: If J = 1, then H = 2 Condition 4: If K = 4, then L = 5 Condition 5: HG

___ ___ ___ ___ ___ ___ ___ G (F) L (F) L H

Fig. 1 Examination of the conditions reveals a few interactions. Per condition 3, if J = 1, then H = 2, and G = 3, which means F = 5. With F on 5, this means that K cannot be on 4. In the alternative, if K = 4, then F = 3. Thus, should condition 4 apply, J cannot be on day 1. These two conditions are mutually exclusive! That helps. Also notice that M, as our free agent, is free to fill any space, since no conditions restrict it. Lastly, remember that conditions 3 and 4 cannot be simply read in reverse. If H = 2, J does not have to = 1. The proper means of reversing it is: if H ≠ 2, then J ≠ 1. 1. This is an elimination question! Whenever there is no new information, simply eliminate the choices that violate the conditions. Hint: To save time, do not compare each choice to each condition. Take each condition, one by one, and look at each choice for the member that violates that specific condition. In nearly every case, each condition will be violated by one answer choice. Here, as there are five conditions and only five answer choices, only four conditions can be violated. If you get confused or loose track of which conditions you have checked, then work through the conditions starting at the top and eliminate those answer choices that violate it. Then each subsequent condition has fewer choices left to examine. (A) Nope, this violates condition 2 by having L on day 6. (B) Sorry, but F cannot be on day 4, condition 1. (C) No, G must come after H, otherwise condition 5 is violated. (D) L is on day 1, but K is on day 4. This violates condition 4. (E) * Correct! This order is possible! 2. Place J on 1. Next, examine the conditions that effect J. Starting at condition 3, if J is on day 1, then H must be on day 2. This places G on day 3. Now, going back to the conditions, only day 5 remains for F. A quick day count reveals that L is forced to day 7, and, to keep from violating condition 4, K must be on day 6. This leaves M to fill the last space, day 4. (Figure 2)

J H G M F K L 1 2 3 4 5 6 7

Fig. 2

Page 9: 1 1 1 1 1 1dl.keywin.org/f/6/f6aa09cee96d90f76619d51be7e69eba.pdf · 2012-04-16 · Get Prepped! PrepTest 27 3 revenue is bankrupting the most popular magazines. But readership is

Get Prepped! PrepTest 27

9

Now, the question was: who must be on day 4? (A) No, this outright violates condition 1. (B) As G must immediately follow H, G must be on day 3. (C) If K is on day 4, then L must be on day 5. However, this doesn�t leave F anywhere to go as G is on day 3. (D) Sorry, but you must have missed condition 2. L can never be on day 4 or 6. (E) * Yes, Moses works on day 4. 3. If K is on day 4, L must be on day 5, per condition 4. This forces F to be on day 3. As 3, 4, and 5 are assigned, this leaves 1 & 2 and 6 & 7 as the remaining pairs for HG to occupy. Thus, J cannot be on day 1, otherwise condition 5 will be violated, as G and H must be together. (A) Sorry, but this violates condition 5. (B) As the only spaces left are 1 & 2 and 6 & 7, this would force the HG pair to 6 & 7. Thus M would have to come before J. (C) * Correct! If J is on 6, M can be on 7. This leaves 1 & 2 for H & G. (D) No, this means J is on day 1. M�s placement would therefore violate condition 3. (E) Sorry, but this would either split H & G or would force J after M. 4. If H is on day 2, then G is on day 3, and F must be on day 5. As such, K cannot be on day 4, as that would force L and F to be on the same day. (A) No, K absolutely cannot be on day 4. (B) While M can be on day 4, so can others. (C) * Correct, J does not have to be on day 1. (D) No, K cannot be on day 4 in this case. (E) Again, K�s presence creates a conflict between conditions 1 and 4. 5. The new conditions are KH and LF. Where does this leave J? Well, to start with, remember condition 5. Thus we actually have KHG and LF as sets. Notice that condition 3 cannot be allowed to happen in this case! As F can only be on 3 or 5 and L cannot be on day 4, LF must be days 2 & 3 respectively. As for the other set, space requirements demand that KHG come after day 3. However, if K is on day 4, condition 4 is violated. Thus KHG must be days 5, 6, & 7 respectively. (Figure 3)

__ L F __ K H G 1 2 3 4 5 6 7

Fig. 3

(A) Tempting, but this would violate condition 3. (B) & (C) No, condition 2 says L can�t be on day 4. Thus F must be on day 3. (D) * Finally. Day 4 is the correct answer. As the diagram shows, if J can�t be on day 1, day 4 is the only choice left. (E) Nope, K must be here to meet the space requirements as well as condition 4. 6. If G and L are before F, then who can be on day 3? Starting with G, HG must be together. Thus, F cannot be on day 3, there are not enough spaces to fit HG and L. So, F must be on day 5. Also, K cannot be on day 4. What now? Let�s plug a few things in and see where they go. If J was on day 1, HG would be days 2 & 3 respectively. This would leave L on day 5, in violation of condition 2. Okay so that doesn�t work. If the rules aren�t revealing anything, lets approach the problem from a spatial �occupancy overlap� point of view. If there are 4 spaces before F, then HG can be on days 1 & 2, 2 & 3, or 3 & 4. Remember that the issue is who can be on day 3. If anyone but H or G is on day 3, the last two options are eliminated, leaving only days 1 & 2. Now this is fine if the person on day 3 is L, but that is not an option. (A) No, as G, H, & L must all be before F, there are not enough spaces left for F to be on day 3. (B) * Yes, this fits the spacing requirements above, and this allows L to be on day 1. (C) � (E) Sorry, but all of these would force HG to days 1 & 2, leaving L for day 4, which violates condition 2.

Page 10: 1 1 1 1 1 1dl.keywin.org/f/6/f6aa09cee96d90f76619d51be7e69eba.pdf · 2012-04-16 · Get Prepped! PrepTest 27 3 revenue is bankrupting the most popular magazines. But readership is

Get Prepped! PrepTest 27

10

Overview of puzzle set #2: Five habitats house 4 snakes and 3 lizards, which consist of 5 females and 2 males total. This initially may sound like a matrix, but then how would we account for the 5 cages? Let�s just stay flexible and see what the conditions indicate. Condition 1: 2 animals max per habitat Condition 2: No mixing species Condition 3: No FS next to ML The two animals per cage condition is interesting. Also, the �immediately next to�� language is interesting. Both of these textual clues indicate a multiple line. We still have no anchors. We can however say that both species require a minimum of 2 habitats because of the spacing requirements. So that leaves one box that may contain either species. Notice that we can�t just assign 1 & 2 to snakes, 3 & 4 to lizards, and so on, since physical order matters. The importance of physical order definitely makes this a line problem. It is tempting to assume that placing 1 animal in each habitat leaves us with 2 animals to pair up with someone, but notice that this would presume there are animals in every habitat. So this doesn�t help us much. However, it does tell us that we don�t need to track species separately within habitats. Therefore we need to track the species and the number of each sex in each habitat. (Figure 1) 4-S 5-F 3-L 2-M __|__ __|__ __|__ __|__ __|__ 1 2 3 4 5 Fig. 1 7. Elimination game! If you have problems with the first question of a set, double check the conditions, as you have likely missed something. (A) No, this violates our no-mixing-of-the-species rule (condition 2). (B) * Correct, there are 4 snakes, 3 lizards, 5 females, and 2 males, and there are no species/gender placement conflicts. (C) FS in H2 next to ML in H3 is not allowed. (D) Whoops, this one has 3 animals in habitat 5. Overcrowding is not allowed. (E) Sorry, we know how much you hate doing math, but there are too many males in this setup. Notice that the correct answer has an empty habitat. No matter how tempting it is to read too much into the problems, if it is not stated, it is not a condition! 8. If H2 has at least 1 FS and H4 contains 2 ML, then the rest of the animals must be females (3 snakes and 1 lizard, for those of you keeping track at home). Remember that each species requires a minimum of two cages, but a FS can�t be next to a ML. This means that the H3 and H5 cannot be used to house the other FS. Therefore, both H1 and H2 both must contain 2 FS! (Figure 2) 4-S 5-F 3-L 2-M (all Ms have now been used) __|__ __|FS __|__ ML|ML __|__ Now, let�s apply condition 3. __|__ __|FS __|__ ML|ML __|__ no FS no FS therefore, need 3 more FS in 1 & 2 therefore, FL must be in 3 or 5. Fig. 2

Page 11: 1 1 1 1 1 1dl.keywin.org/f/6/f6aa09cee96d90f76619d51be7e69eba.pdf · 2012-04-16 · Get Prepped! PrepTest 27 3 revenue is bankrupting the most popular magazines. But readership is

Get Prepped! PrepTest 27

11

(A) & (B) Sorry, remember that FS can�t be next to ML, but there aren�t 2 FL left to use. (C) This

forces a FS into a habitat next to the MLs, as H2 can only house one more FS. (D) Condition 2 forbids mixing of species. (E) * Yes, the remaining FL must be in either H3 or H5. 9. Which must be true? As the answer choices are all math-oriented, you must think in terms of the mathematical relationships that are found in the conditions. What are we given? Well, at most two reptiles can be in a cage and there are 4 snakes, 3 lizards, 5 females, and 2 males. Starting with the gender numbers, as there are an odd number of females, at least one female must either be alone or paired with a male. Turing to the species number that are given to us, there are an odd number of lizards and we can�t mix species, at least one lizard must be by itself. (A) Nope, it can be paired with a male so long as condition 3 is not violated. (B) No, he can be alone, with another male, or with a female. (C) * Correct, the only possibilities are either each lizard by itself or 2 together and the other alone. (D) No, there is nothing that says there can�t be 2 MS. (E) Remember this is a �must� be question. We had 2 ML in the last problem! 10. Another elimination question. This one is harder as it requires you to extrapolate the remaining habitats. (A) This has 2 males and condition 3 doesn�t apply. So this is fine. (B) No problem here; condition 3 only cares if there are FS next to the ML. (C) This works, since it leaves 3 S and 1 L, 2 of which are males. So we can still comply with condition 3. (D) * Well, this doesn�t work, there are only three remaining habitats and there are 3 S and 3 L left. This requires a minimum of 4 habitats! (E) No worries mate, the remaining 4 S and 2 L can be divided among the remaining 3 habitats. 11. H3 is empty, leaving only 4 habitats. No problem, we require a minimum of 4 cages to hold 7 reptiles, but condition 3 still must be met. However, the 2 snake habitats can�t be next to each other; therefore, this requires that a snake and a lizard habitat be so arranged. To uphold condition 3 then, the males cannot be lizards, or they would unavoidably bump into a female snake! Since the 4 Ss have to be split-up, they must be on 1 or 2 and 4 or 5. (A) * This COULD be false. Habitats 1 & 5 are left and they aren�t next to each other. (B) True! No lizard can be male, but the question is asking for what can be false. (C) Only 4 habitats are left, and, as both species require 2 habitats, there must be 2 snakes in each of their habitats. Could be false and �no� equal must be true. Tricky, isn�t it? (D) True! The question requires that snakes not be next to each other. So, as we can�t mix species, the lizards can�t be next to each other either. (E) No, this must be true. There are 4 habitats and 7 reptiles. As the max is 2 per habitat, there must be 3 that contain 2 reptiles and 1 that contains a sole reptile. (Figure 3) 4-S 5-F 3-L 2-M __|__ __|__ X|X __|__ __|__ Fig. 3 12. For this problem there are 4 FS and, therefore, 2 ML and 1 FL. The lizards all get to live alone. This leaves 2 habitats for the 4 FS. The 2 habitats containing the ML cannot be next to the snakes. So we need to determine what are the possible/impossible combinations. Doing it by hand would take forever, so let�s start by thinking it through first. The habitat to either side of the males cannot contain snakes. Thus if a ML is in H2, H1 and H3 are off limits. It should quickly become clear that the habitats with the ML must be together and at one end of the row. Otherwise there will not be enough habitats remaining to house the snakes. Don�t believe me? Put the MLs in H1 and H3, H2 and H4 are off limits, leaving only 1 habitat for the snakes when 2 are required. So, the ML must either be in H1 & H2 or H4 & H5. Both ways, H3 can contain no snakes. (A) & (B) No, if the ML are in H4 & H5, this is fine. (C) * Correct, not matter where the ML are, H3 cannot contain snakes, as they are all females. (D) & (E) No, if the ML are in H1 & H2, this is fine.

Page 12: 1 1 1 1 1 1dl.keywin.org/f/6/f6aa09cee96d90f76619d51be7e69eba.pdf · 2012-04-16 · Get Prepped! PrepTest 27 3 revenue is bankrupting the most popular magazines. But readership is

Get Prepped! PrepTest 27

12

Overview of puzzle set #3- Film Buffs Seven film buffs each see a movie. They have three movies to chose from, F, H, or K. Look at the conditions before choosing the diagram. Condition 1: 2H/F Condition 2: G ≠ R Condition 3: I ≠ M Condition 4: V = Y Condition 5: L = H Condition 6: G = F/K

X X

G I L M R V Y X F H K (H=2xF)

All the conditions focus on who may or may not see movies with each other. This is the hallmark of the subset. Looking at the conditions, only conditions 2 & 6 have people in common. However, as R can still see F, H, or K, depending upon what G watches, this not really helpful. Condition 1, however, does have some hidden information. As there are only 7 people, the number that watch H/F can only be 2/1 or 4/2. Any more than this and it would add up to more than 7! Is there more? Yes, but we�ll work on it later. 13. This question was very hard. The solution to this one was to see how the F and H limits came into play. Do the math. We have 7 people. If 1 person sees F, then 2 people must see H, therefore leaving 4 people to see K. If 2 people see F, then 4 people must see H, therefore leaving 1 person to see K. F must have one or two people. We can eliminate (A) because G can�t see H. (B) we can eliminate, because if V goes to F, then Y would have to go to F, but since I is also going to F, this would be three people, and this is too many. That was pretty complex. (C) If V and Y go to F, then 4 people must go to H. and the only four left are G, I, L, and M. But I and M can�t go together to a movie. (D) * If M, R, and V (and thus Y) go to K, then I and L can go to H, and G can go to F, and this actually works. (E) If M and R (and also L) go to H, then we need one more person to go to H, but G is not allowed, and I can�t because M is going. Definitely a very hard question. It was very easy to get lost in that one. 14. What can be true? (Notice that �must be false except� is the same as �can be false/true,� not �must be true�.) (A) * Yes, if R is the only one to see F, there can be 2 people who see H and 4 that see K. For example, we can have: H= L/M and K = G/I/V/Y. (B) No, there must be at least 2 people who see H. (C) No, if V sees a film, Y must also see that film. (D) No, as either 3 or 6 people must see the H & F films, then K must have either 4 or 1 viewer. (E) As there must be exactly 2 H viewers per F viewer, H must have 2 or 4, and can never be an odd number. 15. Who might not see the H film? (A) & (B) Well, if 2 people don�t see the H film, then 5 would have see it. This isn�t possible under condition 1. (C) * Yes, this would leave L/M/V/Y to see H, two people to see F, and one person to see K. (D) Careful, this should be a complete list, but Y is here without V. (E) Tempting, but realize that this leaves I & M together to see H.

Page 13: 1 1 1 1 1 1dl.keywin.org/f/6/f6aa09cee96d90f76619d51be7e69eba.pdf · 2012-04-16 · Get Prepped! PrepTest 27 3 revenue is bankrupting the most popular magazines. But readership is

Get Prepped! PrepTest 27

13

16. If 1 person sees K, then 4 must see H and 2 must see F. Since V & Y must be together, they cannot see K, nor can they see F. Why? Because this leaves either G & R together or I & M together. (A) * Yes, V & Y must see H! (B) No, G & R are interchangeable here, and G can see either F or K. (C) & (D) No, since I & M must see different movies, but having no other limitations, they are interchangeable. (E) Sorry, but G and R are interchangeable here; thus R can see either F or K. 17. No new information means that the base diagram and the conditions should answer the question. (A) No, there is no restriction as to the relationship between G and I. (B) Nope, there is nothing that precludes I from seeing the H film. (C) Can�t I see H with V, Y, and L? (D) Remember that I & M are interchangeable in this set, as they must be different. (E) * This must be true! L must see H, and G can only see F or K. 18. Finally, new information! Apply the conditions to the information to determine what is possible. Note that as the question asks for which one could be true, the rest must be false. If G and V see the same thing, Y must as well. G can only see F or K, but at most two people may see F. Thus, G, V, & Y must see K. Now, this leaves four people. Per our starting math, this means 2 must see H, 1 must see F, and the other must join G, V, & Y in seeing K. (A) Sorry, but if G watches F, then V & Y must do also. However, that would be three people watching F, and, per condition 1, 6 people watching H. We don�t have that many people. (B) * Yes, I can see H. (C) As G must see K, condition 2 says that R cannot. (D) No, V and G are together, so this would violate condition 6. (E) No, this would force V and G to also watch F, and at most 2 people are allowed to watch F. 19. Who cannot see F? Remember that at most 2 people can see F. So, as long as the combination doesn�t force 2 people together who can�t be, then it is fine. (A) � (D) None of these violate conditions 2 or 3 directly or indirectly. (E) * Correct. If V & Y see F, then either G & R or I & M will be forced to see H together.

Page 14: 1 1 1 1 1 1dl.keywin.org/f/6/f6aa09cee96d90f76619d51be7e69eba.pdf · 2012-04-16 · Get Prepped! PrepTest 27 3 revenue is bankrupting the most popular magazines. But readership is

Get Prepped! PrepTest 27

14

Overview of puzzle set #4- Colorful Cars We have six cars, two each of three different colors, to arrange in a straight line. This sounds like a job for a line diagram. (Figure 1)

___ ___ ___ ___ ___ ___ 1 2 3 4 5 6 Fig.1 Condition 1: Neighbors must be different colors Condition 2: 5 or 6 = P Condition 3: 1 ≠ O Condition 4: 4 ≠ G ___ ___ ___ ___ ___ ___ O G (P) (P)

There is not much in the way of extra warranted conclusions. Car 1 is either P or G; car 4 is either O or P; and condition 2 leaves only 1 P car for the first four places. 20. Who cannot be the same color as each other? Well obviously any set next to each other is out, but this is not one of the answer choices, too bad. Therefore, we need to consider the other limiting factors in the conditions. Only condition 2 actually places a color, even if it is indefinite. (A) * Yes, because of condition 3 and 4, the only way these can be the same color is if they are P. However, this doesn�t leave a P for the 5th or 6th positions. (B) � (E) These can all be the same color. Even if one of the positions in controlled by conditions 3 or 4, none of these combinations excludes compliance with all of the conditions. 21. If 2 and 4 are the same color, which must be false? Car 4 can only be P or O, per condition 4. As 1 P must be saved for 5 or 6, 2 and 4 can only be the same color if they are O. Thus the remaining positions must be either P or G. (A) No, this position can only be either P or G. (B) * Correct, as there is only one P to be used for the first four positions and condition 4 limits the 4th position to O or P, cars 2 and 4 can only be the same color if they are O. (D) � (E) These can be either P or G. The conditions do not set any of the remaining positions to only one of these two colors. 22. If 4 = P, then cars 1 � 3 must be either O or G, as the remaining P is reserved for either position 5 or 6. Now, application of condition 1 forces car 1 to be G. This of course sets car 2 to O and car 3 to G, as condition 1 forbids two of the same color from being next to each other. (Figure 2)

G O G P O P 1 2 3 4 5 6 Fig. 2

(A) No, this violates condition 1. (B) Actually, as car 1 must be G, car 2 cannot be without violating condition 1. (C) Sorry, this must be G. (D) No, this must be O. (E) * Yes! If car 4 is purple, then condition 1 forbids car 5 from also being purple. Notice that choices (A), (B), and (C) simply alternated the cars. This should have been a hint that these were not the correct answer choice, as if one had to be true, then they all did! (Remember that car 4 was fixed as P at the start.)

Page 15: 1 1 1 1 1 1dl.keywin.org/f/6/f6aa09cee96d90f76619d51be7e69eba.pdf · 2012-04-16 · Get Prepped! PrepTest 27 3 revenue is bankrupting the most popular magazines. But readership is

Get Prepped! PrepTest 27

15

23. Which must be false? Start by eliminating those we have used already, then consider the remaining choices. (A) * This can NOT be true. If car 2 is G, then 1 must be P. This uses the available P for the first four spaces. Now condition 1 forces 3 to be O, as no P�s are left, but 4 cannot be G. So this option fails! See figure 3. (B) This was used in problem 21. (C) Sorry, but as long as 4 ≠ P, 5 can. (D) & (E) No, if 5 is set as P, then 6 must be either G or O. P G O ? ___ ___ O G (P) (P) Fig. 3 24. Now we have three G cars and one O. It is very rare to alter the staring conditions. In fact, it will only be done when the initial diagram was too easy. Since there are only six spaces, condition 1 requires that we alternate the placement of the G cars, so the G cars will be 1, 3, and 5/6 or 2, 4 and 6. Now, since 4 ≠ G, then the 1, 3, and 5/6 pattern is the only one that works. Do you see why it can be 5 or 6? Since 1 must be G and 4 cannot be, to fit three Gs, 1 & 3 must be G. However, if 4 is not P, then 5 can be P and 6 can be G! So the alternation feature doesn�t need to carry through! How can you think of this during a test? Well, remember that the point is that the Gs can�t be right next to each other, but neither 5 nor 6 have a G immediately next to them! Thus either can be G. (A) A good start, but this forgets that condition 4 results in 3 also being G. (B) Sorry, but you missed the fact that three G cars coupled with 4 ≠ G forces G into 1. (C) Clearly you were guessing if you got this, as it is the only odd number that doesn�t have to be G. (D) * Yes, 1 and 3 must be G, and 5 or 6 can be. (E) Very tempting, especially when you are short on time, but the test-takers who chose this one got locked-in to thinking that G had to be placed only on the odd numbered spots.

Page 16: 1 1 1 1 1 1dl.keywin.org/f/6/f6aa09cee96d90f76619d51be7e69eba.pdf · 2012-04-16 · Get Prepped! PrepTest 27 3 revenue is bankrupting the most popular magazines. But readership is

Get Prepped! PrepTest 27

16

(292-299)

3 3 3 3 3 3 Overview of the entire section-

This section was a fairly average LSAT reading comprehension section. In all there were twenty-six questions; nine inferences, eight details, six main ideas, and three miscellaneous questions. The first three passages all had seven questions, while the fourth had only five. The first passage, about mass media coverage of criminal court cases, was the easiest. The second passage, about the semantic content of Native American names, was the second most difficult. The third passage, about theories to explain how homing pigeons navigate, was the second easiest of the passage. The fourth passage, about Freudian interpretation of fairy tales, was the most difficult passage. The obvious conclusion you should draw is that doing the passages in order, from first to fourth, was the most efficient order. Note that sometimes the most difficult, or fewest question, passages are the first passages. One strategy is to do the passages with the fewest questions after you have done the other passages. Overview of passage #1- Mass media coverage on criminal trials and juror impartiality

This passage was about average for the LSAT in terms of difficulty and the easiest of the entire section. The subject matter, mass media coverage of criminal court cases and its effects on juror impartiality, is a common topic. The author presented his or her argument in a straightforward manner and did not use complex terms. The passage had seven questions; three main ideas; three inferences; and one miscellaneous. On the exam, because this passage is the easiest and it has seven questions, you would want to answer it first. Here, as it happened, it was first, so even test-takers without the correct strategy still benefited. 1. Main Idea- Main idea questions are the most commonly asked and easiest to answer type of question. To answer a main idea question, you must identify the central, most important, message from the passage� the main idea, the primary purpose, the best title, the best description, or the best summary. The correct answer best summarizes the passage in one sentence. It may be helpful to think about the passage like a news story and pay attention to whom, where, why, what, and when. Be careful, some of the answer choices will accurately describe some aspect of the passage, or a portion of the main idea, but still not entirely capture the main point of the passage. Once you identify the main idea you are able to eliminate answer choices that focus only on specific information. (A) This misrepresents the passage and it includes information not found in the passage. According to the passage, to abandon traditional judicial remedies would not solve the problem of juror impartiality. The passage did not refer to the implementation of new methods. (B) This answer looks promising, but, it is an attractive wrong answer. It accurately restates a fact from the passage (some countries have abandoned traditional methods), but it does not express the main point of the passage. (C) This is an incorrect answer; it accurately restates part of the passage, but it also includes information from outside the passage. According to the passage, voir dire is the most popular method among judges, however; the passage did not identify voir dire as being the most flawed. (D) This is, at best, a minor issue. (E) * This answer is correct; it contains all the major elements: mass media, problem of impartiality, and redefining impartiality. 2. Inference- This question defies easy categorization. The question stem closely resembles a detail question, but, because the answer ultimately comes from outside the passage, it is an inference question. Based on the context in which the author placed the term �mental contortions,� make an inference about the critic�s belief. (A) * This answer is correct. According to the passage, the critic claims that jurors cannot be relied on to ignore information learned outside the courtroom. Therefore, it is logical to conclude that the critic believes that jurors cannot be expected to deliberate on only what they learn at the trial. (B) This answer looks promising, but it is an attractive wrong answer. The passage did not discuss criticism of jurors� inability to distinguish the actual facts of a case. (C) The passage did not discuss criticism of jurors� inability to keep an open mind. (D) Although jurors� inability to identify the degree of prior

Page 17: 1 1 1 1 1 1dl.keywin.org/f/6/f6aa09cee96d90f76619d51be7e69eba.pdf · 2012-04-16 · Get Prepped! PrepTest 27 3 revenue is bankrupting the most popular magazines. But readership is

Get Prepped! PrepTest 27

17

knowledge was mentioned, it was mentioned in the next paragraph and had nothing to do with �mental contortions.� (E) The passage did not mention that jurors should be expected to protect themselves from pretrial publicity. 3. Miscellaneous/Organization- Identify the primary purpose of the third paragraph and keep in mind how it fits into the overall scheme. What does the author use this paragraph to accomplish? (A) This is an incorrect answer; the third paragraph did not mention a new method. (B) * This answer is correct; the primary purpose of the third paragraph was to discuss voir dire and to discuss critics� reasons as to why it is an unreliable method. (C) This is an incorrect answer; the third paragraph only discussed voir dire. (D) This is an incorrect answer; the third paragraph discussed why critics are wary of voir dire. Judges like it. (E) This is an incorrect answer; the third paragraph presented the views of those who believe that judges cannot ensure impartiality using voir dire. 4. Inference- One of the most common type of question, inference/implication questions appear about 20% of the time. Inference questions are typically the most difficult because, unlike a detail question, the answer is not expressly stated in the passage. These questions require you to reason your way to a logical conclusion on the basis of the facts, rather than just finding the facts. The correct answer will be a logical assumption about something from outside the passage that is premised on information found within the passage. Based on the facts, the author would most likely agree with which of the following? (A) This is an incorrect answer; it is not likely that the author would agree with it. In the passage, the author stated that eliminating voir dire is not a solution to the problem of impartiality. Therefore, it unlikely that the author would agree with the notion that flaws in voir dire cause juror impartiality. (B) The author is unlikely to agree. In the passage the author argues that impartiality does not reside in the mind of any one juror. It results from the deliberative process. Therefore, it is incorrect to assume that the author would be likely to agree with this answer. (C) * This answer is correct. The author argued that even opinionated people, with prior knowledge and exposure to media coverage of criminal cases, may be part of an impartial jury. Therefore, it is likely that the author would agree with this answer. (D) This is an incorrect answer; it is unlikely that the author would agree, see (C). (E) Although opinions and knowledge of the facts were discussed separately, the passage provided no basis to assume that the author would agree with this statement. 5. Inference- What does the passage suggest, hint, or allude to as a potential benefit of mass-media coverage on court cases? This is a bit difficult to find, so be creative. (A) This is an obviously incorrect answer; it cites a potential harm of mass-media coverage on court cases. The passage made no suggestion that the mass media will benefit court cases by taking over the role of judge. (B) This answer looks promising, but it is an attractive wrong answer. To minimize juror bias would be beneficial; however, the passage did not suggest media coverage will do this. (C) * This answer is correct; the passage suggested that exposure to mass media is part of being an informed citizen. Informed citizens are necessary to empanel a truly impartial jury; therefore, exposure to mass-media coverage it is part of empanelling a truly impartial jury. See how we assembled the two factual parts to make the inference? (D) This is an incorrect answer. True, judges may change the methods used to question potential jurors because of mass-media coverage; however, that is not beneficial. (E) The passage did not allude to this. In fact, exposure to mass media coverage would likely decrease jurors� awareness of their degree of bias. 6. Main Idea- Which of the following principles most closely reflects the passage�s argument? (A) This actually contradicts the passage�s argument. The passage argued that individual juror�s could bring their personal experiences, including exposure to mass media, with them. (B) * This answer is correct, it is most in keeping with the passage�s argument. Lines 51-55. (C) This is an incorrect answer; it contradicts lines 54-55. (D) This is totally wrong. The passage argued that jurors should not be selected based on their degree of exposure or lack of exposure to mass-media coverage. (E) This principle is unrelated to the passage�s argument. The passage focused on the selection of jurors and pre-trial media coverage, not

Page 18: 1 1 1 1 1 1dl.keywin.org/f/6/f6aa09cee96d90f76619d51be7e69eba.pdf · 2012-04-16 · Get Prepped! PrepTest 27 3 revenue is bankrupting the most popular magazines. But readership is

Get Prepped! PrepTest 27

18

coverage while the trial is in progress. 7. Main Idea- Which of the following reflects the author�s primary purpose in writing the passage? Remember, an author�s primary purpose is generally synonymous with the passage�s main idea. (A) The author did not offer a compromise between proponents and critics of voir dire. (B) This answer looks promising, but it is wrong. While the passage dealt with the effects of mass media on court proceedings, it was not primarily aimed at the effects. (C) Nothing in the passage was explicitly addressed to judges. (D) The author is not a proponent for voir dire, so he definitely is not debating its� critics. (E) * The passage begins and ends by discussing impartiality. It is somewhat tenuous to say the passage argued for a change in how to address impartiality, unless the author is arguing that opinionated people should be sought after and placed on juries.

Page 19: 1 1 1 1 1 1dl.keywin.org/f/6/f6aa09cee96d90f76619d51be7e69eba.pdf · 2012-04-16 · Get Prepped! PrepTest 27 3 revenue is bankrupting the most popular magazines. But readership is

Get Prepped! PrepTest 27

19

Overview of Passage #2- Personal names and semantic content This passage was slightly more difficult. The subject matter, interpretation of personal names with an emphasis on semantic content, is alien to most people. The passage had a total of seven questions; three inferences, two main ideas, one detail, and one miscellaneous. Using your strategies, you should have answered this passage third. 8. Main Idea- Which statement most accurately summarizes and reflects the passage�s main point? (A) This answer appears correct, but it is, in fact, wrong. While the passage stated that Hopi names often perform both functions simultaneously, it is not the main point of the passage. (B) This is an obviously incorrect answer; it misstates facts from the passage (see lines 12-16). (C) This is an incorrect answer; it does not reflect the passage�s main point. It may be true that persons who lack knowledge of Hopi are unable to discern the name�s deeper significance, but it is not the main point of the passage. (D) This is an obviously incorrect answer; it includes information not found in the passage and contradicts parts of the passage itself, it is not even close to being the main idea. (E) * This answer is correct; it accurately and completely captures the main point of the passage, as summarized in lines 15-20. The last sentence of the first paragraph is always a good place to look for the main idea. 9. Inference- Infer the most likely reason that the author referred to Western Apache place names? Recall, the author referred to Western Apache place names in the context of examining Hopi personal names. (A) While it is possible that the author would offer such an example, it is not the most likely reason for the reference in line 46. (B) * This answer is correct; it is most likely that the author referred to Apache place names to apply a commentator�s characterization to Hopi personal names (see lines 44-47). (C) This is incorrect; the author was interested in the similarities between the Apache and Hopi names. (D) This answer looks promising, but it is an attractive wrong answer. While the reference tended to demonstrate that other names may have semantic content, it is not the most likely reason that the author referred to Apache names. (E) This is an incorrect answer; there was no evidence in the passage that a specific Hopi name refers to a particular Western Apache place name. 10. Detail- Detail questions are common, if you had no already figured that out. Because they ask about a particular piece of information, the correct answer is expressly stated in the passage. It is waiting for you to refer back and grab it. Sometimes, a question will reference a line number or provide a term in quotations from the text; in such instances, find the line reference or quote and read the sentence(s) before and after it. Which of the following is an example of Levi-Strauss�s view? Refer to lines 7-10 for the relevant specific information. (A) According to the passage, Strauss viewed names as instruments of social classification. This answer is not an example of a name as an instrument of social classification. (B) Looks or personality do not equate to social classification. (C) Closer, but naming a child after a famous person does not indicate ones social status. (D) This is an obviously incorrect answer; it implicitly contradicts Strauss�s view. (E) * If the name derives from ones occupation, then the name, at one time, was a way to classify that person by their profession. 11. Miscellaneous/Organization- About 25% of the time, you will encounter a miscellaneous type question- a grab bag questions about physical organization, analogy, or attitude. Physical organization questions are concerned with the mechanics of a passage. By marking the passage�s main ideas, important arguments and transition terms, this provides clues to the structure of the passage. When a question asks about the organization of the passage or a paragraph, quickly refer to your notes for an idea of how the passage or paragraph was set up. What is the second paragraph mainly about and how does it fit into the overall passage? (A) This is an incorrect answer; the second paragraph, by itself, presented no reasons why Hopi personal names can only be treated as poetic compositions. (B) This is an incorrect answer; it misstates a fact from the second paragraph. Hopi names do not reference events in the recipient�s life; rather, they refer to the name giver�s clan (lines 24-27). The events are the time when the names are conferred. (C) Neither the second paragraph, nor the facts cited, were intended to refute European theories about naming. (D) * This answer

Page 20: 1 1 1 1 1 1dl.keywin.org/f/6/f6aa09cee96d90f76619d51be7e69eba.pdf · 2012-04-16 · Get Prepped! PrepTest 27 3 revenue is bankrupting the most popular magazines. But readership is

Get Prepped! PrepTest 27

20

is correct; the second paragraph illustrated ways in which Hopi names may have semantic content. (E) This is an incorrect answer; in no way did the second paragraph argue that translations obscure the meaning. 12. Inference- With which statement is the author most likely to agree? Mill felt that names were merely for identification. (A) * This answer is correct; based on evidence in the passage, it is most likely that the author would agree with this statement (see lines 10-14). (B) This is an obviously incorrect answer; based on evidence from the passage, the author would disagree with this statement, it incorporates Strauss�s view. (C) This is an obviously incorrect answer; the author cited both Mills� and Strauss�s views as causing scholars to neglect how names are used outside Europe. (D) & (E) Both Strauss� and Mill�s views are equally relevant to the Hopi names, since Hopi names perform these two functions, as well as be poetic. 13. Inference- This question is awkwardly worded to confuse you, but it is essentially the same as all other inference questions. The only difference is that, in this instance, the correct choice is the one which you cannot assume contributes to Hopi personal names� poetic quality. (A) This is an incorrect answer; It is safe to assume that this feature would contribute to the names� poetic quality. (B) This is an incorrect, see line 44. (C) See line 53. (D) * This is the correct answer; based on the passage, you cannot infer that this feature would contribute to the names� poetic quality. The ability to confer identity on individual is a feature of Mills� characterization of personal names. (E) See paragraph 3. 14. Main Idea- (A) The author did not present an anthropological study of Hopi names. He only mentioned a few examples of names. (B) The author did not propose a new theory; rather, the author examined the inadequacy of Mill�s and Strauss�s theories. (C) This answer looks promising, but it is an attractive wrong answer. The author described three theories of names; however, the author�s primary purpose was to critique two theories by comparing them to a third. (D) * This answer is correct: the author focused on criticizing Mill�s and Strauss�s views. (E) This answer looks promising, but it is an attractive wrong answer. While the author did explain the origin of Hopi Indian names, there was no discussion of cultural origins.

Page 21: 1 1 1 1 1 1dl.keywin.org/f/6/f6aa09cee96d90f76619d51be7e69eba.pdf · 2012-04-16 · Get Prepped! PrepTest 27 3 revenue is bankrupting the most popular magazines. But readership is

Get Prepped! PrepTest 27

21

Overview of passage #3- Homing pigeons This passage was about average for the LSAT in terms of difficulty and the second easiest of the section. Its subject matter, theories explaining how homing pigeons navigate, is relatively easy to understand. This passage also had seven questions; four details, two inferences, and one main idea. On the exam, you would want to answer this passage second. 15. Main Idea- State the passage�s main idea. (A) This is true, but is minor (line 8). (B) * This answer is correct; it accurately, clearly, and concisely captures the main elements of the passage: mapping, and the remaining uncertainty as to how they map. (C) This is an incorrect answer; it is only partly supported by the passage. While the author mentioned that some experiments have been marked by design flaws, it is not necessarily true that a majority (red-flag word) of the experiments have been flawed, nor is it the passage�s main idea. (D) This is an incorrect answer. This is an inference based on the passage, but it is not the passage�s main idea. (E) This is an obviously incorrect answer; it contradicts the passage. In the passage, the author expressly stated that it is unlikely that pigeons have an inertial system (line 16). 16. Detail- This question asks you which of the following is ordinarily true regarding how pigeons �home.� The answer is expressly stated in the first sentence of the passage (lines 1-5). (A) This is an incorrect answer; it is not expressly stated in the passage. (B) * This answer is correct; it is an accurate restatement of the passage. The passage stated that pigeons are �able to choose fairly and accurately homeward bearings within a minute and fly home.�(lines 3-5). (C) This is an incorrect answer; it is not stated in the passage. (D) This is an incorrect answer; it is not stated in the passage. (E) This is an obviously incorrect answer; it contradicts line 5. 17. Detail- At first glance, this question appears different, but is just a detail question (nothing to worry about). The key is to know what the author meant by the term �possibility,� and then to choose the answer that reflects that definition. The �possibility� that the author mentioned in line 6 is the theory that pigeons may find their way home by reading minds. (A) * This would test the possibility that pigeons are able to read the mind of the experimenters. (B) The display of affection is not a variable that would affect the pigeons� ability to read minds. (C) This is incorrect for the same basic reason as (B). (D) This is an incorrect answer for the same reason as (B). (E) This would not effect the birds if they do read minds. 18. Detail- Specific information from the passage supports which of the following? The �first alternative� mentioned in line 16 refers to the theory that pigeons keep track of their outward displacement (see lines 7-10). (A) This contradicts the passage. The first alternative seems unlikely, but it has not been conclusively ruled out. (B) This is an incorrect answer; the passage was silent on whether there are theoretical models. (C) * This answer is correct; it is directly supported by information from the passage. (D) The passage stated that experimental results show that it is not likely correct. (E) The passage did not explicitly mention the difficulty in designing experiments, and it suggested that they are easy to design. 19. Inference- Infer the reason why the author referred to �the system of many short-ranged species� (lines 9-11). (A) The author referred to the pigeon�s ability as �remarkable�; therefore it is illogical to conclude that the ability is universal. (B) * This answer is correct; displacement theory is one of the two basic explanations for the homing ability. (C) This answer looks promising, but it is an attractive wrong answer. The author did state that the explanation seems improbable; however, that was not the reason why the author referred to it. The discrediting is done later (line 16). (D) This is an incorrect answer; the author did not criticize, in this part of the passage, the techniques used by scientists. (E) This is an incorrect answer; the author merely stated the theory in lines 9-11 and later dismissed it. 20. Detail- Based on the passage, which of the following, if true, most weakens Papi�s theory? Locate specific information from the passage about Papi�s theory. Recall, Papi theorized that the pigeons� map

Page 22: 1 1 1 1 1 1dl.keywin.org/f/6/f6aa09cee96d90f76619d51be7e69eba.pdf · 2012-04-16 · Get Prepped! PrepTest 27 3 revenue is bankrupting the most popular magazines. But readership is

Get Prepped! PrepTest 27

22

sense is olfactory (lines 36-37). (A) This is an incorrect answer; if true, it neither strengthens nor weakens Papi�s theory. (B) This is an incorrect answer; if it were true, it would strengthen Papi�s argument. (C) Close, but if true, it would neither strengthen nor weaken Papi�s argument. The fact that pigeons from the same roost take different return routes does not necessarily mean that the pigeons are navigating differently. (D) * This answer is correct; if it were true, it would weaken Papi�s theory. Once the pigeons are beyond the range of odors, they must use something other than their olfactory sense to find their way home. (E) This would have a neutral effect. 21. Inference- Based on the passage, which would the author and Papi both most likely agree? (A) This is an incorrect answer; the author is not likely to agree. The author cited experiments that tend to refute the olfactory theory. (B) This is an incorrect answer; the author dismissed this theory. (C) * Based on the passage, it is likely that both the author and Papi would agree that map sense, in some form, is responsible. (D) This is an incorrect answer, there is no basis to assume that the author would agree with it. (E) This is an obviously incorrect answer; the experiments conducted by Schmidt-Koenig and Phillips cast doubt on Papi�s theory.

Page 23: 1 1 1 1 1 1dl.keywin.org/f/6/f6aa09cee96d90f76619d51be7e69eba.pdf · 2012-04-16 · Get Prepped! PrepTest 27 3 revenue is bankrupting the most popular magazines. But readership is

Get Prepped! PrepTest 27

23

Overview of passage #4- Freudian interpretation of fairy tales This passage was more difficult than an average passage, and the most difficult of the section. Its subject matter, reductive Freudian interpretations of fairy tales, is bizarre, or at least it was for me. The author used jargon and presented the passage in a convoluted manner, perhaps even deliberately so. The passage had only five questions; three details, one inference, and one miscellaneous. On the exam, you would definitely want to attempt this passage last. One thing to note is that if you had previewed the questions, you would have seen that four questions concerned Bettelheim, and one concerned Freud. Knowing this, you could skim the complex discussion of Freud, which was over half the passage. Many test takers sunk into the quicksand of the first paragraph and never escaped. 22. Detail- According to the author, Bettelheim believes that �the fairy tale is therapeutic because children find their own solutions...� (lines 48-50). (A) Escapism was never discussed. (B) * This answer is correct; it restates what the author wrote about Bettelheim�s beliefs. (C) This is an incorrect answer; it was not explicitly stated in the passage. (D) This is an incorrect answer, it was not explicitly stated in the passage. (E) This is an obviously incorrect answer; common sense alone dictates that you reject this answer outright. 23. Detail- According to the passage, why does Bettelheim believe that parents telling fairy tales is therapeutic? It is right there, in black and white. (A) This is an incorrect answer; it was not explicitly stated in the passage. (B) This is an obviously incorrect answer; it was not explicitly stated in the passage. (C) * This answer is correct; it restates the passage. The passage stated that, �By telling the child such stories themselves... parents impart to the child their approval of the stories.� (lines 42-45). (D) This is an incorrect answer; it was not explicitly stated in the passage. (E) This is an incorrect answer; it was not explicitly stated in the passage. 24. Inference- Infer the reason, based on the passage, that Freud believed that in fairy tales, �nothing is incredible� (line 24). (A) This is an incorrect answer; common sense alone dictates that you reject this answer. (B) Close. There is a fine distinction between purely imaginary (not reality-based) and �everything is possible� (anything can happen). (C) * This answer is correct; it is a conclusion that is supported by the evidence in the passage. Freud stated that, in fairy tales �everything is possible, so nothing is incredible.� (D) This is obviously incorrect. Freud specifically excluded fairy tales from the realm of the uncanny (a concept I still don�t understand). (E) There is little or no reference to personal judgment. 25. Detail- According to the passage, Bettelheim believes that children find their own solutions to inner conflicts when they interpret a story benignly (lines 48-54). (A) * This answer is correct; it accurately restates the passage. (B) This is just a mish-mash of words taken from the passage. (C) This is an obviously incorrect answer; it contradicts an explicit statement in lines 52-53. (D) Any such awareness is never mentioned. (E) This answer looks promising, but it is an attractive wrong answer. According to the passage, children are reassured when parents read the story, not when the children benignly interpret the story. 26. Miscellaneous/Attitude- Describe the author�s stance, beliefs, feelings, outlook, view, opinion, or values regarding an element from the passage. If the author were on a TV show, what would he have to say, and how would he or she come across? Describe the author�s attitude towards Bettelheim�s work. The author referred to Bettelheim�s book as a �splendid achievement� (lines 32-33). This fact is an important clue to recognizing the author�s attitude towards Bettelheim�s work. (A) This is factually inaccurate. Bettelheim did not reject orthodox and reductive Freudian interpretations of fairy tales. (B), (C) & (D) If the author were appalled with, unimpressed by, or skeptical of, the work, he would not have referred to it as a splendid achievement. (E) * This answer is correct; the author showed his attitude by expressing an appreciation of the work.

Page 24: 1 1 1 1 1 1dl.keywin.org/f/6/f6aa09cee96d90f76619d51be7e69eba.pdf · 2012-04-16 · Get Prepped! PrepTest 27 3 revenue is bankrupting the most popular magazines. But readership is

Get Prepped! PrepTest 27

24

(300-307)

4 4 4 4 4 4

Analysis of the Questions: 1. Politician- This is a make a conclusion question. You must make a conclusion based on a given set of facts or conditions. Unlike an identify a conclusion type question, the answer is not stated in the passage. When answering these, there are several things to do. Limit your conclusion to the given facts and avoid making assumptions. Do not make a conclusion that is logically possible, but not directly supported by the facts from the passage. Do not select an answer that simply restates a fact or condition (remember, the conclusion is not in the passage). If you find two feasible answer choices, select the one that is narrowest in scope. (A) In this example, the tax would not discourage the continued use of sugar. (B) This answer is obviously incorrect; in this example, the tax will not discourage continued use of the pain relievers. (C) * This answer is correct; it is an application of the politician�s principle. In this instance, the tax on a harmful pesticide is increased for the purpose of discouraging its continued use. (D) This is an incorrect answer; in this example the tax on the harmful substance is not raised, for fear of lowering consumption. If you missed this, it was probably because you got mixed-up. (E) This is an obviously incorrect answer; in this example the tax on a harmful substance is being lowered, not raised. It is hard to see how a lower tax would lead to new uses. 2. The average cable- This is a strengthening question. The LSAT tests this because it is an important skill for law students and lawyers. Your ability to handle these questions efficiently is central to doing well on the test; so practice, practice, and practice them some more. The facts involved are: fiber optics provide 3 times the channels for the same price. Therefore, fiber will replace cable. There are a few assumptions. Let�s find the new fact that will bolster this somewhat presumptuous conclusion. Ignore issues and ideas that are irrelevant to strengthening the argument. Do not try to correct or defend the facts in the passage, take them as true. (A) This answer weakens the argument. If fiber costs more to install, then people will be reluctant to choose it over cable. (B) * This answer is correct; it strengthens the argument. If the programming on fiber optic television is of equal quality to regular cable, then that is one less reason for people not to choose fiber over cable. (C) This answer is incorrect; it weakens the argument. If cable companies increase the number of channels available, then fiber optic television has no competitive advantage. (D) So what? This neither strengthens nor weakens the argument. (E) This answer is incorrect; it weakens the argument. If fiber optic is more regulated, then viewers will have a reason to not switch to fiber, because cable will offer more kinds of programming. 3. A just government- This is an easy make a conclusion question. (A) * This answer is correct; this closely follows the principle cited in the passage. Here, the government allows a person to act upon their desire to own a sports car; however, that right is limited by a restriction (speed limit) intended to protect the health or property of fellow citizens (other motorists). (B) This is way off. It is not possible to reach this conclusion based on the principle cited in the passage. The passage cited a principle of a just government, this example addresses the behavior of an unjust government, services, and labor. (C) This is an example of a limit on government benefits towards its citizens. The principle cited addresses a limit on citizens� actions toward other citizens. (D) This does not follow the principle from the passage. The passage dealt with government limits on the actions of its citizens that pose a threat to other citizens. This is an example of the government limits on actions that pose a threat to the government. (E) This is a matter of degree, and the passage just discussed restricting and not restricting. It didn�t get into questions of degree. 4. Mayor- This is an assumption question. The correct answer will not be explicitly stated within the passage; you must read between the lines. Generally, the passage will contain an explicit conclusion. Here it is that the suggestion is not financially feasible. Now work backwards to reveal the hidden assumption,

Page 25: 1 1 1 1 1 1dl.keywin.org/f/6/f6aa09cee96d90f76619d51be7e69eba.pdf · 2012-04-16 · Get Prepped! PrepTest 27 3 revenue is bankrupting the most popular magazines. But readership is

Get Prepped! PrepTest 27

25

the missing premise, or implicit factual basis. The answer is a logical antecedent, or a necessary pre-condition, of the author�s conclusion. Be leery of an answer choice that merely restates a fact or that introduces a topic wholly unrelated to the conclusion. The Mayor concluded that, because the new asphalt would cost more than plain asphalt, and the budget for roads cannot be increased, the new asphalt is cost-prohibitive. What did the Mayor have to assume to make this conclusion? (A) This answer is incorrect; it is not an assumption upon which the Mayor�s conclusion depends. This example fails to address the issue that is central to the conclusion� whether the savings of the new asphalt outweigh the costs of the old asphalt. (B) This answer is irrelevant. (C) This answer is incorrect. The passage is focused on rubberized asphalt. (D) * This answer is correct; the Mayor had to make this assumption. If the savings of maintaining the new asphalt roads outweigh the costs of installing the new asphalt, then the new asphalt is, in fact, less expensive than the old. To conclude that the new asphalt is more expensive overall, the Mayor had to assume that the savings would be less than the total costs (including maintenance) of the old asphalt. (E) This is a suggestion, not an assumption. 5. Ticks attach- This is a pretty easy make a conclusion question. Select the only answer choice that is the inevitable, logical, conclusion based on the passage. Our facts are clearly listed. We can conclude that since the mice are in their nests when the ticks drop off, that the ticks fall off in the nests. (A) This answer is incorrect. The passage stated that all ticks drop off their hosts between noon and sunset, and that they might attach at any time. (B) This answer is incorrect; based on the passage it is unclear whether this conclusion is valid. Part of the passage supports this conclusion and part of the passage opposes it. Contradictory information is not sufficient to support the correct answer. On the LSAT, any debatable answer is wrong. (C) * This answer is correct; it is the only conclusion that can be drawn with certainty. White footed mice stay in their nest during the day. No matter what, deer ticks drop off the mice during the day. Therefore, deer ticks drop off the mice while they are in the nest. (D) This answer is obviously silly, there is no basis to affirm or deny it. (E) This answer is incorrect; there is no basis to conclude whether it is valid or not. 6. Monarch butterflies- This is a miscellaneous question type. Very few questions do not fit neatly into one of the common formats. When confronted with a miscellaneous problem, pay close attention to the question stem; it will provide insight into how to approach the problem. Each miscellaneous question will be slightly different. The skills you use to answer the common questions also serve well to answer the miscellaneous questions. Here we are asked to figure-out what information would most help us judge the validity of the conclusion. What is the conclusion? That tour groups are endangering the butterflies. The facts are that tourists trample shrubs that fallen butterflies use for protection. What information helps us judge if the conclusion is valid? Easy, how many butterflies fall off?

(A) This answer is incorrect. �Forestland� is irrelevant. It does not provide any information about the tour groups endangering the butterfly population. (B) This answer is incorrect; the information is irrelevant to the tourist conclusion. (C) Vaguely interesting, but incorrect. The information is irrelevant. (D) Since this ignores the tourists, it is irrelevant. (E) * This answer is correct; the information helps us to determine the validity of the conclusion. If the proportion of hibernating monarch butterflies that fall off the trees is large, then the tour groups themselves are endangering the butterfly population. If the proportion is very small, then the groups are not a real threat. 7. If you know- This is a flawed reasoning question. Unlike a weakening the argument question, you do not add a new fact, you merely identify an inherent flaw. There are only a few common types of flaw. You must identify the author�s mistake. It helps to paraphrase the facts and the conclusion. Once in this form, the author�s mistake becomes more obvious. Here, you must identify a key piece of information that the author failed to establish. Let�s paraphrase the passage: If you know about history, then you will impress intellectuals. If you read a lot, then you know about history. If you do not read a lot, then you do not know about history; so, you will not be able to impress intellectuals. The author�s mistake? He or she failed to consider other ways, besides

Page 26: 1 1 1 1 1 1dl.keywin.org/f/6/f6aa09cee96d90f76619d51be7e69eba.pdf · 2012-04-16 · Get Prepped! PrepTest 27 3 revenue is bankrupting the most popular magazines. But readership is

Get Prepped! PrepTest 27

26

knowledge of history, to impress intellectuals. There was another flaw as well. The argument overlooked the possibility that you could know about history without having read a lot. (A) This does not identify the author�s flaw in reasoning. Maybe that�s why they are impressed. (B) The author assumed that any person who knows about history will be able to impress intellectuals; not that they will definitely impress intellectuals. (C) The new subject matter makes this answer obviously incorrect. (D) * This answer is correct; it identifies the flaw in reasoning. Maybe juggling is an easy way to impress them. (E) This answer is incorrect; the author did not consider non-intellectuals. 8. People always seem- This is a resolve the discrepancy question. Find the missing piece of information necessary to explain the apparent contradiction and make both sides correct. Why is crummy tea the same price as good tea? (A) This answer is incorrect; it is neutral. Because the costs of packaging and advertising affect both high and low-quality teas alike, it is not responsible for raising the price of low-quality tea to the level of high-quality tea. (B) * This answer is correct; it explains the counterexample. Surprisingly, the demand for low quality tea is much higher than the demand for high-quality tea. The increase in demand for low quality tea raises its price relative to high quality tea. (C) This answer is incorrect; for the same reason as choice (A); it affects low and high-quality tea the same. (D) This answer is incorrect; if it were true, high quality teas would be much more expensive than low-quality teas. (E) This answer is incorrect; it would have a neutral effect. 9. The only physical- This is fairly hard assumption question. Consider the conclusion. People can travel to Mars with this device and return. The facts are: weight is a problem; carrying enough fuel for the round trip makes the ship too heavy; a device allows us to fill the tank at Mars. Well, I am not sure what the assumption will be, let�s look at the choices. (A) This answer looks promising, but it is an attractive wrong answer. The author explicitly stated that weight is the only physical factor preventing a human journey to mars. The author confines the conclusion to considerations of weight. The actual amounts of fuel are not within the scope of the question. (B) So what? This does not address overcoming the problem of weight (the sole limiting factor). (C) This answer is incorrect for the same reason as choice (B). We care about weight, not space. (D) This answer is incorrect for the same reason as choice (B). (E) * This answer is correct; If the space ship can return to earth, then it must have overcome the problem of weight. If the spaceship has overcome the problem of weight, then the device must weigh less than the current, full, tanks. 10. Unplugging a peripheral- This is a vulnerable to criticism question. This one is vulnerable because it confused necessary and sufficient causes. (A) This answer is incorrect. There was no semantic shift in meaning of the term �unusable.� (B) * This answer is correct; it identifies the mistake. The author failed to consider an alternative explanation for the malfunctioning of the software program. For instance, another peripheral component may be unplugged, or the computer may have a virus. An unplugged mouse is sufficient cause, but it is not a necessary cause. (C) This answer is incorrect. (D) The author did not cite an overly broad generalization. (E) This answer misstates the passage. The passage explicitly stated that the malfunctioning program is one that requires a mouse. 11. P: complying with- This is a point at issue question. This is a rare question type. This question is the first of two questions that follow a dialogue. Frequently, the first question addresses the interaction of the two speakers and the second question addresses only the part of speaker number two. Here, identify the issue in dispute between P and Q. P and Q have each reached different conclusions. It is this difference in their conclusions that is the point at issue. (A) P and Q agree that the fire resulted in costly damage. (B) Neither P nor Q held this position. (C) This answer is incorrect; it does not identify the point at issue. It accurately restates a part of P�s argument, and Q did not disagree. (D) * This answer is correct; it identifies the point at issue. P concluded that to comply with the new regulations is useless, while Q concluded the opposite. Each used their own set of considerations. (E) This answer is incorrect; P and Q did not discuss whether the regulations are likely to be obeyed.

Page 27: 1 1 1 1 1 1dl.keywin.org/f/6/f6aa09cee96d90f76619d51be7e69eba.pdf · 2012-04-16 · Get Prepped! PrepTest 27 3 revenue is bankrupting the most popular magazines. But readership is

Get Prepped! PrepTest 27

27

12. P: Complying with- This is an A responds to B question. Remember the second speaker always disagrees with the first, and we want to know how Q does this. On first glance, it seems that Q brings up new considerations. (A) * This answer is wordy, but correct. P argued that, because they would not have prevented last year�s fire, the new regulations are useless. Q countered by showing that the new regulations would save money, and, in that sense, they are useful. (B) This answer does not describe the method used by Q. Q did not cite evidence to show that the new regulations would have prevented last year�s fire. (C) This answer is incorrect. Q showed how the particular regulations are useful. (D) Q did not attack an assumption of P�s argument. (E) This answer introduces information found nowhere in the passage; neither Q or P mentioned �potential� or �actual benefits.� 13. Historian- This is a parallel reasoning question. One of the ways by which the LSAT tests your ability to evaluate arguments and methods of reasoning is to ask that you mirror a set of conditions. Most likely, you will see only two parallel reasoning questions per section; however, it is important to practice them because they are difficult and unfamiliar. These questions can be time traps for the uninitiated. However, once you have practiced, you will be able to handle them. On the exam, if you are pressed for time, it may be best to attempt to answer these questions last. It is usually easier to identify patterns of reasoning when the arguments are in diagram form. Use words or symbols to place the arguments in diagram form. Be concerned with the blueprint, or design, of the arguments. First, beware of answer choices that deal with the same subject matter, or incorporate the same wording as the passage; generally, they are red herrings. The right answer is not necessarily identical to the passage, but it is the most similar in its reasoning. Here both (C) �laws� and �moral� & (E) �society� and �mistakes� are too similar to the argument. It is unlikely that they are correct. Second. Look for the flaw. Here the flaw is, that if it is wrong for 1 person, it is wrong for the entire collection of persons. Keep this flaw in mind. Now, instead of diagramming the argument, let�s just paraphrase it. It is wrong for one individual to run the government. Therefore it is wrong for the collection of individuals to run the government. (A) Paraphrase this choice: �If he is unable to do it, then they are unable to do it.� This is close, but no dice. (B) This one is not flawed; �he failed in the past, thus he will fail now.� (C) �If it is wrong for one person to do it, it is wrong for a different person to do it.� This moves from individual to another individual. We want a choice that moves from individual to a group made up of individuals. (D) * This answer is correct; it exhibits the same flawed pattern of reasoning: No individual member can afford a tent. The club is made up of individual members. Therefore, the club cannot afford a tent. The flaw should be obvious, they can pool their money. (E) �If all are wrong, then one person is wrong.� This is the inverse of what we wanted. 14. In 1712- This is a make a conclusion question. It is easier than it initially appeared. Use some simple math to find the conclusion. (A) This answer is incorrect; it cannot be concluded from the passage. The passage stated nothing about the number of books published before 1712. Without that information, we can only guess if this is true. (B) This answer is obviously incorrect; it cannot be concluded from the passage. The censors each approved the same total number of books. The first censor rejected 50% of all books, while the second rejected only 25%. Therefore, the first censor prohibited the publication of twice as many books as the second. (C) * This answer is a properly drawn conclusion. Both sensors approved the same number of books. The first censor rejected twice the number of books as the second; therefore more books were submitted to the first censor. Do the math. The first censor approved 6 of 12. The second approved 6 of 8. Ta da! Here is your answer. (D) This choice is speculative, it cannot be concluded from the passage. There is absolutely no basis to support it. (E) This answer is incorrect; it cannot be concluded from the passage. There is no way to tell the number of writers based on the number of submissions; one writer could have written twelve manuscripts and another zero. 15. It is often- This is a weaken the argument type question. It is short, but difficult. (A) This answer is

Page 28: 1 1 1 1 1 1dl.keywin.org/f/6/f6aa09cee96d90f76619d51be7e69eba.pdf · 2012-04-16 · Get Prepped! PrepTest 27 3 revenue is bankrupting the most popular magazines. But readership is

Get Prepped! PrepTest 27

28

incorrect; if true, this fact would tend to strengthen the argument. It suggests that we have independently based our own standard of beauty on objective, universal values. (B) This answer is incorrect; it does not weaken the �importance of the arts� argument; the information is irrelevant. (C) * This answer is correct; it tends to weaken the argument. If true, this fact would suggest that the similarity between our own standard of beauty and the standard of earlier cultures is attributable to the influence of that artwork on us, and not on independent objective values. (D) This answer is incorrect; it does not weaken the argument. We never read about modern views of art. (E) This answer is incorrect; art ownership is wholly irrelevant. 16. Nutrition education- This is a strengthen the argument question with a twist. The correct answer is the lone choice that does not strengthen the argument. The author concluded that promotion of the �four food groups� was dangerous to health. How odd. Which choice either weakens the author�s conclusion, or has a neutral effect on it? (A) If true, this would strengthen the author�s argument. It is evidence that the focus on the four food groups led to over-consumption of harmful fats. (B) If true, this would strengthen the author�s argument. It is evidence that focusing on the four food groups led to under-consumption of beneficial oils. (C) This answer is incorrect; if true, it would tend to strengthen the author�s argument. It is evidence that a focus on the four food groups led to consumption of a single fruit or vegetable, which is insufficient. (D) * This answer is correct; it tends to weaken, not support, the author�s conclusion. It is evidence that focusing on the four food groups led to an increase in the consumption of fruits and vegetables, which is healthy. (E) This answer is incorrect; if true, it would tend to strengthen the author�s argument. 17. The Green Ensemble- This is a make conclusion question. Get the facts in order. Those facts are that the group is dependent on contributions from corporations. Also, the group has been able to continue operations. (A) * This answer is correct; you can properly draw this conclusion from the passage. If any of the corporate sponsors had withdrawn, then, the facts inform us, the ensemble would have disbanded. The ensemble did not disband. Therefore, none of the corporate sponsors withdrew. (B) While this is practical, it contradicts the facts. This conclusion relies on additional information. You must limit your conclusion to the facts from the passage. (C) This answer overreaches; you cannot properly draw this conclusion. The passage suggested that this may be true, but you cannot conclude for certain whether it is or is not. (D) You cannot properly draw this conclusion. There is no factual basis to conclude the percentage of income attributable to corporate sponsorship. (E) This is worse than (C). There is no basis to conclude that funding has increased. The only certainty is that corporate funding has stayed the same. It may or may not have risen. 18. Book publishers- This is a weaken the argument question. The author argued: Because publishers are more interested in making money than publishing unprofitable books of merit, they are publishing fewer books of merit. (A) This answer is incorrect; it tends to strengthen, not weaken the argument. (B) * This new information weakens the argument. It is evidence that the decline in the publication of books of intrinsic merit is attributable to a dearth of quality submissions, not an increased interest in profit. (C) This answer is incorrect; it does not weaken the argument. The fact that, in the past, books of intrinsic have made an unexpected profit is irrelevant to the expectations of today�s publishers. (D) This answer is incorrect; it does not weaken the argument, it has a neutral effect. The motivation of certain authors is irrelevant to the motivation of the publishers. (E) This answer is incorrect; it tends to strengthen, not weaken the argument. It is evidence that the reduction in quality books is due to concerns for declining profit. 19. Most people- This is identify an assumption question and it is more difficult than most. This is one time where eliminating the wrong choices yields the correct one. The author argued that: Because information is either (1) delivered too quickly or (2) it is poorly organized, most people feel they are being confused by the news. People can handle a greater maximum density of information than that of news stories. Therefore, the news is poorly organized. Once, you understand the argument, look for an assumption on which the argument depends.

Page 29: 1 1 1 1 1 1dl.keywin.org/f/6/f6aa09cee96d90f76619d51be7e69eba.pdf · 2012-04-16 · Get Prepped! PrepTest 27 3 revenue is bankrupting the most popular magazines. But readership is

Get Prepped! PrepTest 27

29

(A) * This answer is the correct answer; it is one of the assumptions upon which the argument depends. It seems that the test writers just pulled this assumption out of thin air, but (as you will see through the process of elimination) none of the other answer choices fit. Because the author gave only two possible reasons for people being confused, he or she must have assumed that there was not a third. (B) This answer is incorrect; it is a misstatement of fact, not an assumption. The passage explicitly stated that poor organization of information makes people feel confused. To be confusing is not the same as to be impossible to understand. (C) This answer is incorrect; it is too far off base to be an assumption upon which the conclusion depends. The passage dealt with a problem and its two possible causes. This choice discusses a solution to the problem, not an assumption. (D) This answer is incorrect; it is not an assumption, it�s a fact. The author stated that most people can handle a greater density of information than that of a news story. Exactly how much is that? Is that a very high information density? There is no way to tell. (E) This answer is incorrect; the author explicitly stated that people could handle more information. 20. Art historian- This is an identify an assumption question. The facts are a little quirky. Robbins understands too well to dismiss, but not well enough to praise. (Insert assumption here.) Therefore, Robbins can�t pass judgment. (A) * This answer is correct; it is an assumption upon which the historian�s conclusion depends. Make sure by placing the assumption into the historian�s argument: To judge the art Robbins must be able to praise or dismiss it. Robbins cannot dismiss or praise the art; therefore she cannot judge it. (B) This answer is incorrect; it is not the proper assumption because it is so far off base. The historian reached a conclusion about Robbins in particular. (C) This answer is incorrect; it is not the proper assumption because it mixes the argument. The historian reached a conclusion about what is necessary for Robin�s to judge the art, not to understand the art. (D) This answer is incorrect; it is not the proper assumption because it is too broad. The historian�s argument dealt only with Robbins. Stuart is not the focus. (E) This answer is not the assumption that leads into Robbins passing judgment. 21. Words like- What a funny, and long, argument structure question. Blah, blah, blah. In highly edited form, the argument presented the generalization. Then it presented an apparent counterexample. Then it refined the generalization so as to exclude the counterexample. (A) Maybe, but not. Since the original generalization was not �misanalyzed� this one fails. (B) The argument did not dismiss the counterexample as insignificant. (C) The argument did not say that every generalization has an exception. (D) * This is the correct answer; the argument narrowed the scope of the general rule by adding a qualification. (E) This answer is obviously incorrect; in fact, it is totally incoherent silly-talk. 22. Words like- This is a miscellaneous question type. You will probably never see another question phrased like this. The correct answer, like the counterexample absentee, should be a word, ending in ee, which depicts one person acting by herself. (A) This answer is incorrect. In this instance, one person honors and the other person receives the honor. (B) Here, one person appoints and the other person is appointed. (C) This does not fit the counterexample. One person nominates and the other person is nominated. (D) This answer is incorrect; it does not fit the counterexample. One person transfers and the other person is transferred. (E) * This answer is correct; it is similar to absentee. In this instance, there is only one party to the action, the person who escapes. 23. Much of today�s- This is a make a conclusion type question. There are a lot of facts to keep track of (A) The mention of �significant� makes this initially interesting, but it is incorrect. It cannot be concluded with certainty because it is too broad. The passage compared Hypatia�s latest novel only to her earlier works, not to a majority of today�s literature. (B) Note that this choice does not contain �Hypatia.� Also, this answer is overly broad. The property of grace is mentioned only in connection with Hypatia�s work, not in connection with today�s literature as a whole. (C) This answer is incorrect for the same reason as choice (A). (D) This answer is incorrect for the same reason as choice (A). (E) * This answer is correct; this can be concluded with certainty from the passage. This work exceeds, on at least one of the three qualities, her earlier work. These three qualities are indicative of great literature.

Page 30: 1 1 1 1 1 1dl.keywin.org/f/6/f6aa09cee96d90f76619d51be7e69eba.pdf · 2012-04-16 · Get Prepped! PrepTest 27 3 revenue is bankrupting the most popular magazines. But readership is

Get Prepped! PrepTest 27

30

24. Scientists- This is a tough argument structure. The scientists assume that some penicillin develop resistant bacteria. Then there is a long discussion of how non-penicillin takers may develop resistant bacteria. So what role did the assumption play? That is hard to say. Let�s read the choices. (A) The assumption is neither a hypothesis nor is it proven. (B) This answer contradicts the facts and is obviously incorrect. (C) * This answer is correct; the research stemmed from the need to understand two groups of persons. The first group had developed penicillin resistant bacteria without ever taking penicillin. The second group had developed penicillin resistant bacteria only after taking penicillin. (D) There was no mention of previous research. The assumption was not made false by the mercury discovery. (E) This answer is incorrect; it introduces information that is too far afield, into �genetic makeup.� 25. All any reporter- This is a difficult flaw in the reasoning question. Boy, they are really piling them on at the end. The flaw is already there; you don�t need to add any new facts. There are too many already! (A) Maybe. {This answer is incorrect; it does not point out the flaw in reasoning. It restates an exact sentence from the passage.} (B) No. The author concluded that a reporter could, not that a reporter would, scoop all of the other reporters. This is not the flaw. (C) This does not point out the flaw in reasoning. It contradicts the first sentence of the passage. (D) This does not point out the flaw in reasoning. It is irrelevant whether the press agent knows no more than the most knowledgeable reporter. (E) * This answer is correct; it points out the flaw in reasoning of the conclusion. The author concluded that one reporter can scoop the others. The flaw in this is that the only info the reporters have came from the press agent. It doesn�t matter that the press agent didn�t tell them everything.